Can't do this supremum question

  • Thread starter kramer733
  • Start date
  • Tags
    Supremum
In summary, The question asks to prove that a ≤ b and to conclude that a ≤ sup{a_n: n is in the natural numbers}. The definition of a Least Upper Bound, where b is an upper bound and if y is an upper bound of the set, then y > b or y = b, is given. The question also asks for clarification on the meaning of b and the existence of a limit of a_n. A hint is provided to consider two subsequences, one decreasing and one increasing, to show that the inequality holds for the original sequence.
  • #1
kramer733
323
0

Homework Statement



here's the picture and it's the second part of question 5:

http://imgur.com/ybSW4v4

Homework Equations



N/A

The Attempt at a Solution



so by intuition, I suspect that b = sup{a_n: n is in the natural numbers}

If we can show that, then it will follow from the first part of question 5 that what we are trying to prove does hold.

Ok well the problem is this. I'm given the definition of Least Upper Bound:

1) b is an upperbound
2) if y is an upperbound of the set, then (y > b) or (y = b).

I'm not sure how to prove 2) at all.. I need major help =/ Please..
 
Physics news on Phys.org
  • #2
The question is: $$\text{Q5. }\text{Suppose that }a_n\leq b\text{ for all } n \text{ and that } a=\lim_{n\rightarrow \infty} a_n \text{ exists. }\\
\text{5.1 Show that }a\leq b\text{. }\\
\text{5.2 Conclude that } a\leq \sup_n a_n = \sup_n \{ a_n : n\in \mathbb{N} \}$$
... hmmm, how to describe it...

if A={1,2,3}, then sup[A] = what?,
if an ≤ b, what is b?
Is b supposed to be a single number or a set of them?
What does it mean to say "## a=\lim_{n\rightarrow \infty} a_n## exists"?
 
  • #3
Oh crap... For whatever reason, i parsed "b" to be the number such that it is greater than AND equal to an. Thank you. So b is a set of numbers that satisfy an≤ b right?
 
  • #4
I'm working on this same question. I believe b is supposed to be a real number so that the sequence is bounded.
 
  • #5
Zondrina said:
I'm working on this same question. I believe b is supposed to be a real number so that the sequence is bounded.

I've posted a hint in Zondrina's post; you can check it out there: https://www.physicsforums.com/showthread.php?p=4364873#post4364873

Basic idea: consider two subsequences of your sequence [itex]a_n[/itex], one that is decreasing, and one that is increasing. Show that if you have an decreasing sequence, then your inequality is satisfied. Show that this also holds for increasing sequences. Conclude that this inequality is valid for your original sequence.
 

1. What is a supremum in mathematics?

A supremum, also known as the least upper bound, is the smallest number that is greater than or equal to all the numbers in a set. It is a concept commonly used in analysis, calculus, and other branches of mathematics.

2. How do you prove that a supremum exists?

In order to prove that a supremum exists for a set of numbers, you must show that the set is bounded above and that it has an upper bound that is smaller than any other upper bound for the set. This can be done using various mathematical techniques such as the completeness axiom or the least upper bound property.

3. What does it mean if a set does not have a supremum?

If a set does not have a supremum, it means that there is no smallest number that is greater than or equal to all the numbers in the set. This can occur when the set is unbounded or when there is no number that is greater than or equal to all the numbers in the set.

4. Can a supremum be a negative number?

Yes, a supremum can be a negative number if the set contains negative numbers and the negative number is the smallest number that is greater than or equal to all the numbers in the set. However, if the set only contains positive numbers, the supremum will always be a positive number.

5. How is a supremum different from a maximum?

A supremum is the smallest number that is greater than or equal to all the numbers in a set, while a maximum is the largest number in a set. The supremum may or may not be a member of the set, while the maximum is always a member of the set. Additionally, a set can have multiple supremums, but it can only have one maximum.

Similar threads

  • Calculus and Beyond Homework Help
Replies
1
Views
655
  • Calculus and Beyond Homework Help
Replies
3
Views
1K
  • Calculus and Beyond Homework Help
Replies
8
Views
7K
  • Calculus and Beyond Homework Help
Replies
1
Views
1K
  • Calculus and Beyond Homework Help
Replies
5
Views
2K
  • Calculus and Beyond Homework Help
Replies
7
Views
2K
  • Calculus and Beyond Homework Help
Replies
7
Views
2K
  • Calculus and Beyond Homework Help
Replies
7
Views
2K
Replies
9
Views
890
  • Calculus and Beyond Homework Help
Replies
4
Views
820
Back
Top